Mit der Quantenmechanik solltest du klarkommen. Eventuell brauchst du noch etwas Funktionalanalysis (Operatoren, Hilberträume, Spektralsätze), aber diese sollte in einem Buch über Quantenmechanik auch erklärt werden. Für die Quantenmechanik würde ich mich aber auch in Theoretischer Mechanik (u.a. Hamilton-Funktion) einlesen.

Die spezielle Relativitätstheorie sollte auch nicht schwierig sein, da sie mathematisch recht einfach ist.

Mit der allgemeinen sieht es aber ganz anders aus. Hier wirst du um Differentialgeometrie (Tensoren, (Lorentz-) Mannigfaltigkeit, Geodätische) nicht herum kommen. Wobei ich mich da auch eher an den Matheteilen in Physikbücher orientieren würde, als an reine mathematische Bücher über Differentialgeometrie. So etwas wie De-Rham-Kohomologie wirst du nicht brauchen.

Mit Quantenfeldtheorien kenne ich mich nicht aus. Davor würde ich aber sowieso erst versuchen, Quantenmechanik und Elektrodynamik zu verstehen.

...zur Antwort

Auf welchem Niveau willst du dich den in Physik einlesen? Schulniveau oder eher in den Unistoff? Unabhängig davon würde ich aber mit Mechanik anfangen. Für das Schulniveau kann ich dir Leifiphysik empfehlen. Da sind gute Aufgaben (inkl. Lösungen) und Erklärungen enthalten. Allgemein kannst du dich für die Schule auch an die Lehrpläne in deinem Bundesland (oder einem anderen) richten. Also schau einfach mal da rein, wie da Physik aufgebaut ist.

...zur Antwort

Meiner Meinung nach sollte es stimmen. Ich sehe jetzt keinen Fehler, aber die ... Argumente können natürlich trügen. Für den Fall AB=BA kann man es einfacher ausrechnen:

Dieses Ergebnis kommt dann auch bei deiner Formel raus:

Also ein gutes Zeichen.

...zur Antwort

Habt ihr einen (erlaubten) Spickzettel? Wenn ja, dann würde ich auf keine Fall Teile des Skripts auswendig lernen. Falls nicht, dann kann es nicht schaden die wichtigsten Sätze im Kopf zu haben. Meistens kann man diese aber sowieso schon durch das Durchrechnen von Aufgaben vor der Klausur. Ansonsten würde ich dir einfach empfehlen Altklausuren oder Klausuren aus dem Internet zu rechnen. Vor allem zu Lineare Algebra I und Analysis I gibt es dazu viele mit Lösungen. Mehr habe ich eigentlich nicht gemacht.

...zur Antwort

Hast du schon auf Englisch gesucht? Bei solchen Spezialgebieten wird man auf Deutsch kaum was finden. Ich bin relativ schnell auf das Gebiet der "Gravitational Biology" gestoßen. Vielleicht ist das ja etwas an das deine Suche anknüpfen kann. Ich habe zum Beispiel zwei Bücher zu diesem Gebiet gefunden:

  • Gravitational Biology I
  • Gravitational Biology II

Diese Bücher drehen sich zwar, soweit ich es im Inhaltsverzeichnis erkennen kann, nicht um globale Entwicklungen des gesamten Ökosystem, sondern um die Mikroprozesse in den Pflanzen. Aber eventuell kann man ja vom einen auf das andere schließen. Zudem könntest du in den jeweiligen Quellen weitere hilfreiche Arbeiten finden.

Das Problem wird aber wohl auch sein, an die Bücher zu kommen. Ohne universitären Zugang könnte es da schwierig werden.

...zur Antwort

Hier geht es im Grunde nur darum, die Definition von gleichmäßig stetig stehen zu haben.

Was bei der Abschätzung von

unter anderem gemacht wird, ist x^2 nach oben abzuschätzen. Da x in [1,2] liegt, folgt x^2<=4. Nun will man ganz am Ende stehen haben:

,denn das steht auch so in der Definition von gleichmäßig stetig. Daher muss man den Teil mit sin(x^2/N) noch geeignet abschätzen. Wir versuchen ihn daher gegen ε/4 abzuschätzen, da dann ganz am 4*ε/4=ε stehen bleibt. Genau das was wir wollen.

Im Grunde könnte man auch einfach ε statt ε/4 wählen. Dann hätte man

Daraus würde immer noch gleichmäßige Konvergenz folgen, da ε sowieso beliebig klein werden kann. Aber in der Mathematik mag man schöne Lösungen, daher schätzt man so ab, dass am Ende nur noch ein ε wie in der Definition von gleichmäßig stetig steht.

Noch als Tipp: Oft sieht man vielleicht nicht direkt, was die "schöne" Abschätzung sein soll. Du kannst daher auch einfach mal ein ε' wählen und schauen, wie man das dann besser festlegen kann. Also man wählt das ε' erst nachdem man fertig ist. Im Beweis sieht es dann aber so aus, als wüsste man es a priori schon.

...zur Antwort

Also entweder du kopierst die Argumente der Aufgabe einfach und dann solltest du auch den anderen Fall beweisen können. Alternativ kannst du dir folgendes überlegen. Zuerst sehen wir:



Daraus folgt:



Allgemein gilt zudem det(A)=det(A^T). Insgesamt erhalten wir:



...zur Antwort

Lambda ist eine Diagonalmatrix. Die transponierte Matrix einer diagonalen Matrix ist wieder die gleiche Matrix. Um das zu sehen, nehmen wir einfach mal an, dass A diagonal ist und B=A^T. Dann gilt b_ij=a_ji. Für i ungleich j gilt aber a_ji=0=a_ij, also auch b_ij=0 und damit b_ij=a_ij. Für i=j gilt b_ii=a_ii. Insgesamt gilt also b_ij=a_ij und damit A=A^T.

...zur Antwort

Die Ableitung in 0 ist zwar 0, aber die Begründung ist falsch. Du musst hier die Definition anwenden. Also überprüfen ob folgender Grenzwert existiert:

 Das ist einfach die h-Methode im Punkt 0. Für einen solchen Grenzwert habt ihr vielleicht schon Aussagen bewiesen.

...zur Antwort
Orte eines Gipfels mittels Höhenprofils bestimmen.?

Wußtet ihr, daß Hühnerküken einen Bauchnabel haben?
Tja, was ich nicht weiß ist die Lösung folgender Aufgae

Einen Ansatz muss ich natürlich auch liefern, das ist ja klar

i) Also ich hätte jetzt einfach die partiellen Ableitung nach x,y und z gebildet.

dV/dx = y² + z² 3²

dV/dy = 2yx + e^(-yz) (-z)

dV/dz = 2zx³ + e^(-yz) (-y)

=> ∇ V (x,y,z) = (y² + 3z² x², 2xy+ ze^(-yz), 2zx³ - ye^-yz).

=> d/dt V (x,yz) = dV/dx * dV/dt + dV/dt * dy/dt + dV/dz * dz/dt

= (y² + 3z² x²) dx/dt + (2xy + ze^(-yz) dy/dt + (2zx³ - ye^(-yz)) dz/dt.

Jetzt ist das Problem, daß ich ja noch dx nach dt irgendwie unterbringen muss. Aber es taucht ja nirgendswo im Ausdrucke in "t" auf.
Unsere Tutorin meinte, dass x,y,z ja zeitabhängig sind und darin dann das t auftaucht. Als Hinweis hat sie uns noch an die Tafel geschrieben: dV/dt = dV/dx * dx/dt , wobei dx/dt = x' wär.
Ich bin verwirrt. Ich dachte dv/dx wär die Ableitung nach x.
Mir ist schon bewusst, daß wenn x,y und z zeitabhängig sind, dass dann so in die Richtung "Irgendwas mal t" geht, aber ich weiß nicht, wie ich das mathematisch ausdrücken soll.

ii) dh/dx = exp (-√(x-2,5)²+(y+3)² *(x-2,5)/√(x-2,5)²+(y+3)² - 0,2 (x+1,5)/√(x+1,5)²+(y-2)²

dh/dy = -exp (-√(x2,5)²+(y+3)² * (y+3)/√(x-2,5)²+(y+3)² - 0,2 (y-2)/√(x+1,5)²+(y-2)²

Jetzt wollen die aber bei dem Ort bestimme ne Koordinate hören.
Ich weiß aber nicht, was ich da einsetzen soll.
Google meint, man soll jetzt dh/dx und dh/dy schätzen bis x' = x+ Delta x und y' = y + Delta y und die Abstände infinitesmimal klein werden. Habt dann in dh/dx und dh/dy jeweils für (x,y) = (0,0), (1,1,) (-1,-1) und (3,3) eingesetzt. Kam ((-0,132802 ; -0,1146) , (-1,004 ; ,-0611), (-0,343 ; -0,118) und (0,010591; 2,55163) raus.
Und das ist ja auch nicht der wahre Hugo.

Mit freundlichen Grüßen,
Tobias Heinken

...zum Beitrag
dV/dt = dV/dx * dx/dt

Das stimmt auf jeden Fall nicht. Es gibt zwei Lösungen. Eine lange und eine kürzere. Die lange benutzt, dass V(x,y,z)(t) eine Funktion von R nach R ist, von der du einfach die Ableitung bilden kannst. Wichtig ist auch, dass du die Ableitungen von x,y,z nicht kennst. Daher notiere die Ableitungen davon als die jeweiligen Variablen mit einem Punkt darüber. Wichtig ist auch richtig abzuleiten. Als Beispiel (ich kann hier leider keine Punkte machen. Daher verwende ich '):

Du leitest ja die Funktionen x und y ab!

Die zweite Möglichkeit ist die Kettenregel. An diese wird deine Tutorin auch gedacht haben:

Also die Ableitung nach t entspricht dem Skalarprodukt aus dem Gradienten ausgewertet in t und den Ableitungen der Funktionen x,y,z. (Anm. Die Punkte am Ende sollten Kommas sein).

ii) Weißt du wie man Extremstellen von mehrdimensionalen Funktionen berechnet? Bzw. von Funktionen der Form:

Dazu musst du den Gradienten bilden und dessen Nullstellen berechnen. Also wie bei 1-dim. Funktionen die Ableitung gleich 0 setzen. Dann kann es aber wieder Hoch-, Tief- und Sattelpunkt geben. Die Gipfel sind natürlich Hochpunkte. Um diese bestimmen zu können, musst die die Hesse-Matrix bilden und überprüfen, ob sie in den Extremstellen negativ definit ist. Die Hesse-Matrix ist die höherdimensionale Verallgemeinerung der zweiten Ableitung als Kriterium um die Extremstellen genauer zu untersuchen.

...zur Antwort

Du könntest mal hier reinschauen. Da findest du einige Vermutungen:

https://arxiv.org/pdf/2207.00652.pdf

Offene Probleme aus der Harmonischen Analysis (der direkten Verallgemeinerung der Fourier-Analysis) findest du hier:

https://arxiv.org/pdf/1701.06637.pdf

Es sollte klar sein, dass diese Problem nicht gerade einfach zu verstehen sind.

...zur Antwort

Ich schreibe nun etwas ausführlicher, da ich dein genaues Vorwissen nicht kenne. Unter dem Tangentialraum T_p(M) verschiede ich den Vektorraum bestehend aus Derivationen

Das Tangentialbündel TM ist die disjunkte Vereinigung aller Tangentialräume also:

Ein Vektorfeld X ist eine glatte Abbildung M-->TM mit X(p) in T_p(M). Die Menge aller Vektorfelder bezeichne ich mit:

Das was hier als kovariante Ableitung bezeichnet wird, ist auch bekannt als Levi-Civita-Zusammenhang. Und dieser ist (wie der Name vermuten lässt) ein Zusammenhang. Hier ist das eine Abbildung

mit gewissen Eigenschaften. Eine davon ist die Leibniz-Regel (entspricht der Produktregel bei der gewöhnlichen Ableitung):

Nun zu deiner eigentlichen Frage. In deinem Fall ist:

Mit der Leibnizregel folgt:

Nun müssen wir verwenden, dass gilt:



und, dass ∂_r hier einfach die partielle Ableitung nach r darstellt. Das kannst du dir je nach Definition relativ schnell überlegen. Wir berechnen:

Zusammenfügen liefert nun das Ergebnis im Bild und insbesondere das Umkreiste.

...zur Antwort

Ja, das sollte passen. Das ganze hängt natürlich auch etwas davon ab, wie a) gezeigt wurde. Wenn du z.B. nur

 verwendest, kannst du die a) auch ohne konkrete Angabe von pi_U lösen. Dann muss man in den nächsten Teilaufgaben noch zeigen, dass

  Das hast du aber schon in a) gemacht, muss man jedoch nicht.

...zur Antwort

Linear unabhängig sind Vektoren falls jede Linearkombination der 0 trivial ist. Das heißt du musst folgendes zeigen:

 Warum muss das für alle x gelten? sin und cos sind Funktionen, die hier auf dem Intervall definiert sind. Zwei Funktionen sind gleich, wenn sie in jedem Punkt gleich sind. Also muss die Linearkombination in jedem Punkt gleich 0 sein, da die Null im Funktionenraum die Nullfunktion ist.

...zur Antwort

Ohne dir die Lösung zu sagen, würde ich mir zuerst die Definitionen klar machen. Also was heißt beschränkt, Infimum, usw. Zum Beispiel ist A beschränkt, wenn du reelle Zahlen M und N findest, sodass für alle a in A gilt:

Übersetzt auf die Definition von A bedeutet, dass gelten muss:

Das kannst du nun als erstes versuchen zu zeigen.

...zur Antwort
Ich denke, dass dn zwischen 2 und 4 alterniert

Das stimmt, zumindest nähert sich die Folge abwechselnd beiden Werte an. Das bedeutet also, dass die Folge divergiert. Daher kann sie nicht konvergieren. Die Folge konvergiert nicht gegen 2 und 4. Sie besitzt aber die zwei Teilfolgen d_(2n) und d_(2n+1), die gegen 4 und 2 konvergieren. Falls die Folge d_n konvergiert, müssten alle Teilfolgen gegen den selben Wert konvergieren.

Bei b) sollst du vermutlich benutzen, dass (n/n-1))^n gegen e konvergiert. Oder das vielleicht sogar mitbeweisen. Dafür kannst du den dir bereits bekannten Limes sogar verwenden.

...zur Antwort

Sei

eine Kurve im R^n. Dann gilt:

Dabei verwenden wir zuerst die Kettenregel und anschließend, dass die Jacobi-Ableitung in diesem Fall dem transponierten Gradienten entspricht. Den letzten Term kann man unter der Symmetrie des Skalarprodukts zu dem von dir gesuchten Term umformen. Beachte, dass wir die (totale) Differenzierbarkeit von f voraussetzen.

...zur Antwort